0
$\begingroup$

Let $P$ be a second order elliptic operator defined in a smooth bounded domain $\Omega \subseteq \mathbb{R}^n, n \geq 2$. $\lambda_0$ is the principal eigenvalue of the Dirichlet eigenvalue problem $$ \begin{aligned} & P u=\lambda u \quad \text { in } \Omega . \\ & u=0 \quad \text { on } \partial \Omega . \end{aligned} $$ In [1,2] Donsker and Varadhan generalized the variational formula for general second order elliptic operators with $C^{\infty}$ coefficients. Donsker and Varadhan proved that $ \lambda_0$ is given by $$ \lambda_0=\inf _{\mu \in \mathcal{M}} \sup _{u \in \mathcal{D}} \int_{\Omega} \frac{P u}{u} \mu(d x) . $$ where $\mathcal{M} \equiv \mathcal{M}(\Omega)$ is the space of all probability measures on $\bar{\Omega}$, and $\mathcal{D}$ denotes the set of all positive functions $u \in C^{\infty}\left(\mathbb{R}^n\right)$ for each of which there exist constants $c_1$ and $c_2$ such that $0<c_1 \leq u(x) \leq c_2<\infty$ for all $x \in \mathbb{R}^n$. The proof is based on strongly continuous semigroups theory and was motivated by a probability theory.

Besides, in [3], is was stated that the first eigenvalue is always real, but by the above formula, the space of test function is always strictly positive.

So for non self-adjoint linear elliptic operator, we have a real principal eigenvalue and a corresponding strictly positive eigenfunction? This seems to be a very strong conclusion. If this is true, is this still true on closed manifold?

[1]Donsker, Monroe D., and SR Srinivasa Varadhan. "On a variational formula for the principal eigenvalue for operators with maximum principle." Proceedings of the National Academy of Sciences 72.3 (1975): 780-783.

[2]Donsker, Monroe D., and SR Srinivasa Varadhan. "On the principal eigenvalue of second‐order elliptic differential operators." Communications on Pure and Applied Mathematics 29.6 (1976): 595-621.

[3]Protter, Murray H., and Hans F. Weinberger. "On the spectrum of general second order operators." (1966): 251-255.

$\endgroup$
1
  • $\begingroup$ Something doesn't seem right about the formula for $\lambda_0$ as given in the question. Try it, for instance, for the Dirichlet Laplacian on an interval. One needs to encode the boundary conditions into $\mathcal{D}$ to make it work. $\endgroup$ Commented Nov 2 at 8:24

0

You must log in to answer this question.

Start asking to get answers

Find the answer to your question by asking.

Ask question

Explore related questions

See similar questions with these tags.